usmle board review step 1

357
www.brain101.info Neuroscience Board Review Step I # 1

Upload: pereje70

Post on 10-Apr-2015

1.949 views

Category:

Documents


6 download

DESCRIPTION

review for step 1

TRANSCRIPT

Page 1: usmle Board Review Step 1

www.brain101.info

Neuroscience

Board ReviewStep I # 1

Page 2: usmle Board Review Step 1

2www.brain101.info

The anterior border of the parietal cortexis the:A. parieto-occipital sulcusB. the parieto-frontal sulcusC. the central sulcusD. the lateral fissureE. none of the above

Page 3: usmle Board Review Step 1

3www.brain101.info

C. the central sulcus

Page 4: usmle Board Review Step 1

4www.brain101.info

Which of the following mark the borderbetween the pons and midbrain on thedorsal surface of the brainstem?A. pontomedullary junctionB. exit of the oculomotor nerveC. exit of the trochlear nerveD. superior colliculus caudal edge

Page 5: usmle Board Review Step 1

5www.brain101.info

C. exit of the trochlear nerve

Page 6: usmle Board Review Step 1

6www.brain101.info

The rostral end of the corpus callosum iscalled the:A. GenuB. SpleniumC. BodyD. Anterior Horn

Page 7: usmle Board Review Step 1

7www.brain101.info

A. Genu

Page 8: usmle Board Review Step 1

8www.brain101.info

What separates the parietal and occipitallobes on the medial surface of the brain?A. preoccipital notchB. nothing but an imaginary lineC. the calcarine sulcusD. the parieto-occipital sulcus

Page 9: usmle Board Review Step 1

9www.brain101.info

D. the parieto-occipital sulcus

Page 10: usmle Board Review Step 1

10www.brain101.info

Which of the following neurotransmitters is themain inhibitory neurotransmitter of the CNS?A. Substance PB. DopamineC. AcetylcholineD. EnkephalinE. Gamma Amino Butyric Acid (GABA)

Page 11: usmle Board Review Step 1

11www.brain101.info

E. Gamma Amino Butyric Acid (GABA)

Page 12: usmle Board Review Step 1

12www.brain101.info

Which of the following is NOT acatecholamine neurotransmitters?A. tyrosineB. dopamineC. norepinephrineD. epinephrineE. all are catecholamines

Page 13: usmle Board Review Step 1

13www.brain101.info

A. tyrosine

Page 14: usmle Board Review Step 1

14www.brain101.info

The common enzyme in the synthesis ofall of the catecholamine neurotransmittersis called:A. choline acetyl transferaseB. catechol-O-methyl transferaseC. dopamine -B- hydroxylaseD. tyrosine hydroxylase

Page 15: usmle Board Review Step 1

15www.brain101.info

D. tyrosine hydroxylase

Page 16: usmle Board Review Step 1

16www.brain101.info

Alpha motor neurons contain which of thefollowing enzymes?A. tyrosine hydroxylaseB. choline acetyl transferaseC. dopamine beta hydroxylaseD. catechol -O- methyl transferase

Page 17: usmle Board Review Step 1

17www.brain101.info

B. choline acetyl transferase

Page 18: usmle Board Review Step 1

18www.brain101.info

Which of the following would be useful inreducing skeletal muscle tone?A. nicotineB. muscarineC. curareD. atropine

Page 19: usmle Board Review Step 1

19www.brain101.info

C. curare

Page 20: usmle Board Review Step 1

20www.brain101.info

The gaps in the myelin coating of axonsare called:A. collateralsB. axon hillocksC. nodes of RanvierD. axon boutons

Page 21: usmle Board Review Step 1

21www.brain101.info

C. nodes of Ranvier

Page 22: usmle Board Review Step 1

22www.brain101.info

The gap between neurons is called:A. ephapseB. synapseC. Node of RanvierD. Nothing. There is no gap betweenneurons.

Page 23: usmle Board Review Step 1

23www.brain101.info

B. synapse

Page 24: usmle Board Review Step 1

24www.brain101.info

Dorsal root ganglion cells in the adultPNS are normally of what structural type:A. Purkinje cell typeB. unipolor or pseudounipolarC. bipolarD. multipolar

Page 25: usmle Board Review Step 1

25www.brain101.info

B. unipolor or pseudounipolar

Page 26: usmle Board Review Step 1

26www.brain101.info

Spinal bifida results from:A. failure of closure of the anterior neuroporeB. failure of closure of the posterior neuroporeC. blockage of the ventriclesD. b and cE. all of the above

Page 27: usmle Board Review Step 1

27www.brain101.info

B. failure of closure of the posteriorneuropore

Page 28: usmle Board Review Step 1

28www.brain101.info

Which of the following areas is derivedfrom the rhombencephalon?A. midbrainB. ponsC. medullaD. b and cE. all of the above

Page 29: usmle Board Review Step 1

29www.brain101.info

D. b and c

Page 30: usmle Board Review Step 1

30www.brain101.info

The postnatal growth of the brain from about350 to 1400 grams is primarily due to:A. increased numbers of neuronsB. increased amount of cerebrospinal fluidC. mylenation of axons and development ofblood vesselsD. increased numbers of axons and dendrites

Page 31: usmle Board Review Step 1

31www.brain101.info

C. mylenation of axons and developmentof blood vessels

Page 32: usmle Board Review Step 1

32www.brain101.info

The sinus that directly connects to thejugular vein is the:A. inferior sagittal sinusB. the cavernous sinusC. the transverse sinusD. the sigmoid sinusE. the superior sagittal sinus

Page 33: usmle Board Review Step 1

33www.brain101.info

D. the sigmoid sinus

Page 34: usmle Board Review Step 1

34www.brain101.info

Blockage of the subarachnoidgranulations produces:A. non-communicating hydrocephalusB. communicating hydrocephalusC. spina bifidaD. thrombosis

Page 35: usmle Board Review Step 1

35www.brain101.info

A. non-communicating hydrocephalus

Page 36: usmle Board Review Step 1

36www.brain101.info

The lumbar cistern is a region of:A. the dura materB. the subarachnoid spaceC. the pia materD. the sinuses

Page 37: usmle Board Review Step 1

37www.brain101.info

B. the subarachnoid space

Page 38: usmle Board Review Step 1

38www.brain101.info

Uncal herniation occurs when the uncusof the temporal lobe compresses themidbrain. This occurs when the uncus ispushed:A. into the foramen magnumB. under the tentorial notchC. under the falx cerebriD. rostrally

Page 39: usmle Board Review Step 1

39www.brain101.info

B. under the tentorial notch

Page 40: usmle Board Review Step 1

40www.brain101.info

If a patient showed non-communicatinghydrocephalus with enlarged lateral ventricles,but normal third and fourth ventricles, then thereis most likely blockage in:A. cerebral aqueductB. medial apertureC. lateral aperturesD. foramen of Monro

Page 41: usmle Board Review Step 1

41www.brain101.info

D. foramen of Monro

Page 42: usmle Board Review Step 1

42www.brain101.info

Damage to the level of the spinal cord that liesbelow the L1 vertebra should result in loss ofsensory or motor function:A. on the anterior top of the thighB. on the foot onlyC. the upper and lower limbs on the ipsilateralsideD. on the foot and posterior aspect of the leg,thigh and buttocks

Page 43: usmle Board Review Step 1

43www.brain101.info

D. on the foot and posterior aspect of theleg, thigh and buttocks

Page 44: usmle Board Review Step 1

44www.brain101.info

The dorsolateral motor column of theventral horn is most prominent at the:A. cervical levelB. thoracic levelC. coccygeal levelD. sacral level

Page 45: usmle Board Review Step 1

45www.brain101.info

A. cervical level

Page 46: usmle Board Review Step 1

46www.brain101.info

A positive Romberg sign occurs whenthere is damage to:A. the superior cerebellar pedunclesB. unconscious proprioceptive pathwaysC. the cerebellar hemispheresD. the vestibular nuclei

Page 47: usmle Board Review Step 1

47www.brain101.info

B. unconscious proprioceptive pathways

Page 48: usmle Board Review Step 1

48www.brain101.info

Atropy and Fasciculations in theabdominal muscles just below theumbilicus indicate a lower motor neuronlesion at:A. Upper Cervical levels of the spinal cordB. L1, L2 or L3 levels of the spinal cordC. T6, T7, T8 levels of the spinal cordD. T11 or T12 levels of the spinal cord

Page 49: usmle Board Review Step 1

49www.brain101.info

D. T11 or T12 levels of the spinal cord

Page 50: usmle Board Review Step 1

50www.brain101.info

Which of the following arteries supply thelateral medulla with blood?A. anterior spinalB. basilarC. posterior inferior cerebellar arteryD. anterior inferior cerebellar arteryE. posterior cerebral

Page 51: usmle Board Review Step 1

51www.brain101.info

C. posterior inferior cerebellar artery

Page 52: usmle Board Review Step 1

52www.brain101.info

Unilateral blockage of the posteriorcerebral artery, and the resultingbrainstem damage, often produces a lossof fine touch and pain sensation on thewhich side?A. ipsilateralB. contralateralC. bilateral

Page 53: usmle Board Review Step 1

53www.brain101.info

B. contralateral, due to damage to themedial lemniscus and ALF in themidbrain.

Page 54: usmle Board Review Step 1

54www.brain101.info

Which of the following are NOT a branchof the internal carotid artery?A. anterior cerebral arteryB. middle cerebral arteryC. posterior cerebral arteryD. opthalamic artery

Page 55: usmle Board Review Step 1

55www.brain101.info

C. posterior cerebral artery

Page 56: usmle Board Review Step 1

56www.brain101.info

The anterior inferior cerebellar artery is adirect branch of:A. the vertebral arteryB. the anterior cerebral arteryC. the basilar arteryD. the internal carotid

Page 57: usmle Board Review Step 1

57www.brain101.info

C. the basilar artery

Page 58: usmle Board Review Step 1

58www.brain101.info

Blockage of the paramedian branches of thebasilar artery usually affects the abducensnerve. If the lesion grows laterally, what othercranial nerve is likely to be affected as well.A. facialB. hypoglossalC. vestibulocochlearD. glossopharyngeal

Page 59: usmle Board Review Step 1

59www.brain101.info

A. facial

Page 60: usmle Board Review Step 1

60www.brain101.info

The adult brain is about 2% or so of theadult body. What percent of the bloodpumped by the heart flows to the brain?A. 1%B. 2%C. 15%D. 30%

Page 61: usmle Board Review Step 1

61www.brain101.info

C. 15%

Page 62: usmle Board Review Step 1

62www.brain101.info

Blockage of what artery is likely toproduce damage to the visual cortex,oculomotor nerve and the corticospinaltract?A. superior cerebellarB. anterior cerebralC. middle cerebralD. posterior cerebral

Page 63: usmle Board Review Step 1

63www.brain101.info

D. posterior cerebral.Superior alternating hemiplegia, a.k.a.Weber’s syndrome.

Page 64: usmle Board Review Step 1

64www.brain101.info

What vessel supplies thisgyrus?

A. Anterior cerebral artery.

B. Middle cerebral artery.

C. Posterior cerebralartery.

Page 65: usmle Board Review Step 1

65www.brain101.info

The correct answer is A.The cingulate gyrus is located on themedial surface of eachhemisphere above the corpus callosum,and is vascularized bybranches of the anterior cerebral artery.

Page 66: usmle Board Review Step 1

66www.brain101.info

Which of the following is incorrect in regard tocerebrospinal fluid?A. It is normally clear and colorless.B. Increased glucose concentration is usuallymeaningless.C. Normal concentration of lymphocytes isapproximately 100/mm3.D. Normal specific gravity is approximately1.007.E. Normal pressure is approximately 160 mm.H2O cerebrospinal fluid.

Page 67: usmle Board Review Step 1

67www.brain101.info

C. Normal concentration of lymphocytesis approximately 100/mm3.

Page 68: usmle Board Review Step 1

68www.brain101.info

All of the following are true in regard to spinal corddevelopment except.A. The sulcus limitans delimits the alar from the basalplates.B. Alar neuroblasts are always formed before basalneuroblasts.C. The general visceral afferent and general visceralefferent cell columns lie closest to the sulcus limitans.D. Spinal dysraphism is where there is duplication of thedorsal horns.E. The gray matter of the adult represents the mantlelayer of the embryo.

Page 69: usmle Board Review Step 1

69www.brain101.info

B. Alar neuroblasts are always formedbefore basal neuroblasts

Page 70: usmle Board Review Step 1

70www.brain101.info

All of the following would be found in theKluver-Bucy syndrome except:A. Hyperorality.B. Hyposexuality.C. Psychic blindness.D. Obesity.E. Altered endocrine responses.

Page 71: usmle Board Review Step 1

71www.brain101.info

B. Hyposexuality.

Page 72: usmle Board Review Step 1

72www.brain101.info

A 70 year old male presents with a 2 day history of worseninggeneralized headache and increasing obtundation. He nowcomplains of stiffness in his neck. Vital signs include T 38.7 C,pulse 85, respirations 23, and blood pressure 130/85 mm Hg. ACBC reveals a WBC count of 16,850/microliter. Serum electrolytesreveal a sodium of 145 mmol/L, potassium 4.3 mmol/L, chloride103 mmol/L, CO2 26 mmol/L, urea nitrogen 18 mg/dL, and glucose88 mg/dL. A lumbar puncture yields cloudy cerebrospinal fluid witha glucose of 32 mg/dL, protein 146 mg/dL, and cell count of 3800WBCs (95% PMNs and 5 % mononuclears) and 122 RBCs. Thiscondition has the potential to result (sooner or later) in all of thefollowing complications EXCEPT:A HydrocephalusB Thrombosis and infarctionC AbscessD Subdural hematomaE Cerebellar tonsillar herniation

Page 73: usmle Board Review Step 1

73www.brain101.info

(D) CORRECT. A subdural hematoma isa typical complication of traumatic injury,not bacterial meningitis.

Page 74: usmle Board Review Step 1

74www.brain101.info

A 68 year old woman has been placed in a nursing home by herson because she can no longer be cared for at home. She iscontinually wandering away from the house and getting lost in theneighborhood. She has difficulty keeping her room in order. Shemisplaces articles of clothing and sometimes dresses herself in anodd fashion. These problems have gotten progessively worse overthe past 6 years. She took early retirement as an accountantbecause she was having trouble keeping her clients accounts inorder. There is no history of trauma. She has no history of seizures.Which set of histopathologic findings is most typical for herunderlying disease process:A Atrophy of caudate nucleus and gliosisB Wallerian degeneration and gliosisC Substantia nigra depigmentation and loss of neuronsD Grouped atrophy of muscle with anterior horn cell lossE Neurofibrillary tangles and senile neuritic plaques

Page 75: usmle Board Review Step 1

75www.brain101.info

(E) CORRECT. Plaques and tangles aretypical for Alzheimer's disease, the mostcommon form of senile dementia.

Page 76: usmle Board Review Step 1

76www.brain101.info

During radical prostatectomy, the anesthesiologistreports a drop in blood pressure in a 73-year-old male.The hypotension persists for 30 minutes despiteintervention with pressor agents. The most likelyconsequence of this prolonged hypotensive episode isdevelopment of:A HydranencephalyB Linear parasaggital infarctionC Lacunar infarcts of basal gangliaD Parietal lobe hemorrhagic infarctE Anterior pituitary necrosis

Page 77: usmle Board Review Step 1

77www.brain101.info

(B) CORRECT. The most sensitive areasof the brain to ischemia will be the'watershed' areas between anterior andmiddle cerebral arterial circulations.

Page 78: usmle Board Review Step 1

78www.brain101.info

A 28 year old male undergoes induction chemotherapy for acutemyelogenous leukemia. He becomes severely pancytopenic, with aWBC count of 1320/microliter, Hgb 7.9 g/dL, and platelet count of72,000/microliter. He becomes comatose. A CT scan of the headreveals a right parietal hemorrhage. He dies a day later. Atautopsy, he is found to have a right middle cerebral arterialthrombosis with right parietal hemorrhagic infarction. Which of thefollowing infectious agents is most likely to produce these findings:A Herpes simplex virusB Toxoplasma gondiiC Human immunodeficiency virusD CytomegalovirusE Aspergillus niger

Page 79: usmle Board Review Step 1

79www.brain101.info

(E) CORRECT. Aspergillus loves toinvade blood vessels and producethrombosis. Neutropenia is a risk foraspergillosis.

Page 80: usmle Board Review Step 1

80www.brain101.info

A baby is born prematurely at 29 weeks gestational age to a G2 P1woman whose previous pregnancy resulted in a normal term birth.She reported no difficulties with the current pregnancy, though shecontinued to smoke 1 pack of cigarettes per day. Following thedelivery, the baby receives surfactant therapy and does notdevelop respiratory distress from hyaline membrane disease.However, it is noted that the baby has a seizure during the next dayof life. The most likely pathologic lesion to explain this complicationis:A Intraventricular hemorrhageB Intracerebellar hemorrhageC Subdural hematomaD Subgaleal hemorrhageE Epidural hemorrhage

Page 81: usmle Board Review Step 1

81www.brain101.info

(A) CORRECT. A premature child,particularly in the gestational age rangefrom 22 to 30 weeks, is at risk forgerminal matrix hemorrhage, which oftenextends into the intraventricular region.

Page 82: usmle Board Review Step 1

82www.brain101.info

Following an episode of severe head trauma suffered ina motorcycle accident, an 18 year old female is noted tohave decerebrate posturing. Funduscopic examinationreveals marked bilateral papilledema. A CT scanreveals changes of marked diffuse cerebral edema.This increase in brain volume because of an increase insodium and water content is most likely to be severestin which of the following components:A Neuronal cell bodiesB MeningesC White matterD DuraE Ependyma

Page 83: usmle Board Review Step 1

83www.brain101.info

(C) CORRECT. The greatest amount ofsalt and water increase with cerebraledema occurs within white matter.

Page 84: usmle Board Review Step 1

84www.brain101.info

A 31 year old female has noted recent difficulty with writing. Shehas difficulty controlling her hand movements, and the writing isnearly illegible. A neurological examination reveals decreasedstrength in the right upper extremity and decreased sensation overthe left lower extremity. She has no decrease in mentation, andthere is no reported seizure activity. A lumbar puncture isperformed, and the CSF contains elevated levels of IgG, somemononuclear cells, and oligoclonal bands on gel electrophoresis.Which of the following pathologic findings in the CNS best accountsfor these findings:A Loss of pigmented neurons in the substantia nigraB Perivascular lymphocytes with demyelinated axons in whitematterC Increased neurofibrillary tangles and neuritic plaques in thecortexD Periventricular lymphoid aggregates with cells marking withCD19E Foci of multnucleated cells and macrophages in grey and whitematter

Page 85: usmle Board Review Step 1

85www.brain101.info

(B) CORRECT. The findings point tomultiple sclerosis, which is marked byplaques of demyelination. Theperivascular lymphocytes suggest aninflammatory etiology, but the cause ofthis disease is unknown. MS can presentwith a host of variable neurologicproblems because the plaques ofdemyelination can occur almost anywherein the brain.

Page 86: usmle Board Review Step 1

86www.brain101.info

A 23 year old primagravida has a fetalultrasound performed at 17 weeks gestation.There is no family history of inherited diseases.The pregnancy has been uncomplicated. Whichof the following congenital conditions is themost likely to be seen in this setting:A AnencephalyB EncephaloceleC Arnold-Chiari malformationD LissencephalyE Holoprosencephaly

Page 87: usmle Board Review Step 1

87www.brain101.info

(A) CORRECT. This occurs in about 1birth per 1000. It is usually not associatedwith chromosomal abnormalities. It maybe associated with maternal folatedeficiency.

Page 88: usmle Board Review Step 1

88www.brain101.info

A 28 year old G3 P2 woman has had an uncomplicatedpregnancy. An ultrasound is performed at 16 weeksgestation, and the findings prompt performance of anamniocentesis. The amniotic fluid is found to have anelevated alpha fetoprotein. Which of the followingfindings seen on ultrasound examination of the fetus ismost likely to have been present:A HoloprosencephalyB HydrocephalusC EncephaloceleD Hydrops fetalisE Spina bifida

Page 89: usmle Board Review Step 1

89www.brain101.info

(C) CORRECT. This is a form of neuraltube defect in which the occiput is notformed and herniation of brain occurs.Alpha fetoprotein is increased in amnioticfluid or maternal serum.

Page 90: usmle Board Review Step 1

90www.brain101.info

A 50-year-old African-American male had a bloodpressure of 182/108 mm Hg at the last visit to hisphysician several months ago. He refuses to take anymedications. He is admitted to the hospital aftersuddenly losing consciousness. When he is aroused, hecannot speak and he cannot move his right arm or hisright leg. He probably has:A An embolus to the middle cerebral arteryB A subfrontal meningiomaC Cerebral venous thrombosisD Alzheimer's diseaseE Hemorrhage into the putamen

Page 91: usmle Board Review Step 1

91www.brain101.info

(E) CORRECT. The basal ganglia regionis the typical location for hypertensivehemorrhages.

Page 92: usmle Board Review Step 1

92www.brain101.info

A 52 year old male has a history of chronic alcoholism. He isadmitted after an episode of trauma in which he fell and hit theback of his head. A head CT scan shows no intracranialhemorrhage. He continues to exhibit decreased mentation and abrain MRI scan is performed that shows anterior vermian atrophy ofthe cerebellum. Of the following physical examination findings,which would he most likely have:A Choreiform movementsB NystagmusC Truncal and lower limb ataxiaD Tremor at rest that diminishes or disappears with movementE Short-term memory loss

Page 93: usmle Board Review Step 1

93www.brain101.info

(C) CORRECT. Cerebellar disease oftenmanifests with ataxia.

Page 94: usmle Board Review Step 1

94www.brain101.info

A previously healthy 31-year-old female suddenlyloses consciousness and is taken to the hospitalwhere an emergent head CT scan revealsextensive subarachnoid hemorrhage at the base ofthe brain. She is afebrile. A lumbar puncture yieldscerebrospinal fluid with many red blood cells, but nowhite blood cells. The CSF protein is slightlyincreased, but the glucose is normal. Which of thefollowing conditions do you most strongly suspectthat she has:A Acute bacterial meningitisB Ruptured berry aneurysmC Progressive multifocal leukoencephalitisD Tay-Sachs diseaseE Parkinson's disease

Page 95: usmle Board Review Step 1

95www.brain101.info

(B) CORRECT. About 1% of thepopulation have such an aneurysm. Theycan rupture suddenly.

Page 96: usmle Board Review Step 1

96www.brain101.info

Several members of a large family are affected by theonset of decreasing mental function and motorcoordination when they reach middle age. Theirmovements are marked by choreoathetosis. Genetictesting reveals increased trinucleotide CAG repeats.Which of the following structures is most likely to appeargrossly abnormal at autopsy of the affected persons::A Caudate nucleusB MidbrainC Temporal lobeD Locus ceruleusE Dorsal root ganglion

Page 97: usmle Board Review Step 1

97www.brain101.info

(A) CORRECT. Huntington disease (HD)is inherited in an autosomal dominantpattern. The gene is on chromosome 4,coding for a protein called huntingtin.Normally, there are 11 to 34 copies of theCAG repeat. There are more copies withHD; a greater number of copies correlateswith earlier onset of the disease.

Page 98: usmle Board Review Step 1

98www.brain101.info

A 50 year old male has been imbibing martinis (shaken,not stirred) for several hours while at the blackjacktable. He wanders off, and several minutes later isfound down. Paramedics arrive, and discover a bruiseon his posterior occiput, but no other signs of trauma.He is transported to the hospital in stable condition, withvital signs showing blood pressure 115/80 mm Hg,temperature 36.5 C., pulse 81, and respirations 20. Onarrival, his blood ethanol is 330 mg/dL. He becomesprogressively obtunded. His right pupil is 8 mm and theleft 4 mm. A head CT scan reveals a collection of bloodin the right subdural region. Damage to which of thefollowing structures has resulted in these findings:A Middle meningeal arteryB Cavernous sinusC Middle cerebral arteryD Dural bridging veinE Great vein of Galen

Page 99: usmle Board Review Step 1

99www.brain101.info

(D) CORRECT. Falls in older persons,with some degree of cerebral atrophy canresult in tearing of the bridging veins,which bare more exposed, and lead todevelopment of a subdural hematoma.Such hematomas can develop over avariable length of time.

Page 100: usmle Board Review Step 1

100www.brain101.info

Enlargement of the cerebral ventricles fromobstruction to the flow of cerebrospinal fluid isLEAST likely to occur from which of thefollowing conditions:A Previous meningitis from PneumococcusB Forking of the aqueduct of SylviusC Epidural hematomaD EpendymomaE Intraventricular hemorrhage

Page 101: usmle Board Review Step 1

101www.brain101.info

(A) CORRECT. Postmeningitic statesmore typically lead to communicatinghydrocephalus from deficient reabsorptionof CSF.

Page 102: usmle Board Review Step 1

102www.brain101.info

A 48 year old woman has developed chronic renalfailure, and a renal scan shows bilaterally enlargedkidneys with multiple cysts. She has the suddenonset of a severe headache. A cerebral angiogramdemonstrates marked narrowing of cerebral arterybranches near the base of the brain, consistent withvasospasm, but no intraparenchymal hemorrhageis present. Which of the following conditions mostlikely produced these findings:A Bacterial meningitisB Severe atherosclerosisC Malignant hypertensionD Cerebral edemaE Subarachnoid hemorrhage

Page 103: usmle Board Review Step 1

103www.brain101.info

(E) CORRECT. The blood irritates the arteries.This effect is often delayed by several daysfollowing the initial hemorrhagic event. Berryaneurysms are located in the circle of Willis atthe base of the brain, and rupture leads toextravasation of blood into the subarachnoidspace. Her renal scan suggests a diagnosis ofdominant polycystic kidney disease (DPKD).About 10% of persons with DPKD develop berryaneurysms.

Page 104: usmle Board Review Step 1

104www.brain101.info

A 41 year old woman had a worsening headache for thepast week, along with a fever and increasingobtundation,. A head CT scan reveals a solitary 3 cmdiameter lesion with ring enhancement located in theright parietal lobe. A stereotactic biopsy is performedand a frozen section shows granulation tissue withadjacent collagenization, gliosis, and edema. Theprobable cause for these findings is:A Chronic brain abscessB AspergillosisC Progressive multifocal leukocencephalopathyD ToxoplasmosisE Rabies

Page 105: usmle Board Review Step 1

105www.brain101.info

(A) CORRECT. Granulation tissue withfibrosis is a typical reaction to a cerebralabscess. Collagen deposition around aring enhancing lesion is typical for anabscess that organizes. A commonsource for such a brain abscess is a lunginfection.

Page 106: usmle Board Review Step 1

106www.brain101.info

A 66-year-old male is finding that he has more difficultymoving about. He is annoyed by a tremor in his hands,but the tremor goes away when he performs routinetasks using his hands. His friends remark that he seemsmore sullen and doesn't smile at them, but only stareswith a fixed expression on his face. He has not sufferedany loss of mental ability. Which of the followingdiseases is he most likely to have:A Amyotrophic lateral sclerosis (ALS)B Alzheimer's diseaseC Parkinson's diseaseD Niemann-Pick diseaseE Tuberous sclerosis

Page 107: usmle Board Review Step 1

107www.brain101.info

(C) CORRECT. The tremor at rest istypical for Parkinson's disease. A 'mask-like' facies is another manifestation of thisdegenerative disease resulting from lossof pigmented neurons in the substantianigra.

Page 108: usmle Board Review Step 1

108www.brain101.info

A 72 year old female has a five year history ofworsening mental functioning with trouble rememberingthings. She has no problems with movement. She isnoted on an MRI scan of the brain to havesymmetrically increased size of the lateral ventriclesalong with cerebral cortical atrophy in a mainly frontaland parietal distribution. A lumbar puncture reveals anormal opening pressure, and analysis of the clear,colorless cerebrospinal fluid reveals a glucose andprotein which are in normal ranges. Cell count on theCSF shows 3 WBCs (all lymphocytes) and 1 RBC. Afunduscopic examination is normal. Which of thefollowing findings is most likely associated with herunderlying disease process:A Loss of gamma aminobutyric acidB Presence of the e4 allele of apolipoprotein EC Increased numbers of Lewy bodiesD Perivascular mononuclear inflammationE Loss of Betz cells

Page 109: usmle Board Review Step 1

109www.brain101.info

(B) CORRECT. She has findingscharacteristic for Alzheimer's disease.The neuritic plaques have amyloid cores,and patients may also have a congophilicangiopathy.

Page 110: usmle Board Review Step 1

110www.brain101.info

A 26 year old previously healthy woman has the suddenonset of mental confusion. She has a seizure and isbrought to the hospital. Her vital signs show bloodpressure 100/60 mm Hg, temperature 37 C., pulse 89,and respirations 22. A lumbar puncture reveals anormal opening pressure, and clear, colorlesscerebrospinal fluid is obtained with 1 RBC and 20WBC's (all lymphocytes), with normal glucose andprotein. An MRI scan reveals swelling of the righttemporal lobe with hemorrhagic areas. Which of thefollowing infectious agents is the most likely cause forthese findings:A Herpes simplex virusB Influenza virusC Mycobacterium tuberculosisD Hemophilus influenzaeE Neisseria meningitidis

Page 111: usmle Board Review Step 1

111www.brain101.info

(A) CORRECT. Hemorrhagic lesions of thetemporal lobe are typical for Herpes simplexvirus infection. Affected persons do not have tobe immunocompromised. (Note: when thispatient was brought to the ER, the examiningphysician initially passed the problem off as a'drug overdose' which was his default diagnosisfor any mental problem in a young person, butthe family refused to accept that and pressedhim for further workup).

Page 112: usmle Board Review Step 1

112www.brain101.info

Which is NOT a pain sensitive structure:A. ScalpB. Brain parenchymaC. Trigeminal nerveD. Cranial sinusesE. 3rd cervical nerve

Page 113: usmle Board Review Step 1

113www.brain101.info

The correct answer is B.The cranial structures that are insensitiveto pain include: Parenchyma of the brain,Ependyma and choroid plexus, Pia mater,arachnoid membrane and parts of thedura mater, and the skull (periosteumslightly sensitive.)

Page 114: usmle Board Review Step 1

114www.brain101.info

A 45 year old man complains of nocturnal severeheadaches which typically awaken him from sleepabout two hours after going to bed. He states that thepain is intense and most frequently on the right side ofthe head. His wife has noted some flushing of that sideof the face. The headaches are 20 minutes in durationand occur repeatedly for weeks at a time and thenabate.The most likely diagnosis is:A. Ophthalmopelegic migraineB. Brain TumorC. Tension headacheD. Cluster headache

Page 115: usmle Board Review Step 1

115www.brain101.info

The correct answer is D.These are unilateral headaches almost alwaysoccurring the same side associated withflushing, sweating, rhinorrhea, lacrimation,ptosis, and occasionally Horner's syndrome. Itoccurs most frequently in men and lasts 10minutes to 1 hour. It occurs in clusters (severalheadaches a week with long periods of time inbetween in which the patients are headachefree.

Page 116: usmle Board Review Step 1

116www.brain101.info

A 45 year old man complains of nocturnal severeheadaches which typically awaken him from sleepabout two hours after going to bed. He states that thepain is intense and most frequently on the right side ofthe head. His wife has noted some flushing of that sideof the face. The headaches are 20 minutes in durationand occur repeatedly for weeks at a time and thenabate.The workup and treatment should be:A. Intravenous compazineB. MRI scan as soon as possibleC. A routine CT scan and treatment with amitriptylineD. Ergotamine and/or Oxygen

Page 117: usmle Board Review Step 1

117www.brain101.info

The correct answer is D.Treat with sub-lingual, inhalational, orintramuscular Ergotamine, Oxygen,Lidocaine nose drops, and/orSumatriptan.

Page 118: usmle Board Review Step 1

118www.brain101.info

Absence Seizures (Petit Mal) arecharacterized by which of the following:A. Post ictal confusionB. Loss of postural controlC. Typical duration of 1 to 2 minutesD. All of the aboveE. None of the above

Page 119: usmle Board Review Step 1

119www.brain101.info

The correct answer is E.Absence seizures involve a abrupt loss &return of consciousness (brief), oftenwithout major motor component (eg. eyeblinking). It can occur hundreds per daywithout recognition. Onset usually inyoung children.

Page 120: usmle Board Review Step 1

120www.brain101.info

Match the epilepsy type with thetreatment:Idiopathic genralized tonic-clonic epilepsyA. ACTH and/or vigabatrinB. Valproic acidC. Phenytoin or Tegretol or Valproic acidD. Ethosuximide or Valproic acidE. Felbamate or Clonazepam

Page 121: usmle Board Review Step 1

121www.brain101.info

The correct answer is C.Carbamazepine will also work.

Page 122: usmle Board Review Step 1

122www.brain101.info

Match the epilepsy type with thetreatment:Juvenile myoclonic epilepsyA. ACTH and/or vigabatrinB. Valproic acidC. Phenytoin or Tegretol or Valproic acidD. Ethosuximide or Valproic acidE. Felbamate or Clonazepam

Page 123: usmle Board Review Step 1

123www.brain101.info

The correct answer is B.

Page 124: usmle Board Review Step 1

124www.brain101.info

Match the epilepsy type with thetreatment:Childhood absence epilepsyA. ACTH and/or vigabatrinB. Valproic acidC. Phenytoin or Tegretol or Valproic acidD. Ethosuximide or Valproic acidE. Felbamate or Clonazepam

Page 125: usmle Board Review Step 1

125www.brain101.info

The correct answer is D.

Page 126: usmle Board Review Step 1

126www.brain101.info

A patient with Wernicke's Encephalopathyis likely to have which of the followingclinical findings:

A. AtaxiaB. NystagmusC. Bilateral 6th nerve palsiesD. All of the aboveE. None of the above

Page 127: usmle Board Review Step 1

127www.brain101.info

The correct answer is D.Clinical findings include Oculomotor signs(Nystagmus, Bilateral 6th nerve palsies -weakness/paralysis of abduction,horizontal diplopia, internal strabismus, &abnormal response to caloric stimulation),Global confusional-apathetic state,Memory disorder (Korsakoff’s), & ETOHwithdrawal.

Page 128: usmle Board Review Step 1

128www.brain101.info

In Alzheimer's Disease the amount ofatrophy seen on CT or MRI scan stronglycorrelates with the degree of dementia.A. TrueB. False

Page 129: usmle Board Review Step 1

129www.brain101.info

B. The correct answer is False.Degree of atrophy inconsistently relatedto degree of dementia.

Page 130: usmle Board Review Step 1

130www.brain101.info

Which are treatable causes of dementiaA. Hypothyroidism (myxedema)B. Creutzfeldt-Jakob diseaseC. B12 deficiencyD. All of the aboveE. A & C only

Page 131: usmle Board Review Step 1

131www.brain101.info

The correct answer is E.Hypothyroidism treatment consists oflevothyroxine. B12 deficiency treatmentcalls for regular intramuscular B12.

Page 132: usmle Board Review Step 1

132www.brain101.info

In a patient with suspected multiple sclerosiswhich finding(s) in the spinal fluid would causeyou to consider alternative diagnoses:A. Protein greater than 100 mg/dlB. Oligoclonal bands are presentC. Cell count greater than 50 per cubic milliliterD. A and CE. None of the above

Page 133: usmle Board Review Step 1

133www.brain101.info

The correct answer is D.In MS, serum electrophoresis is normal:Total CSF protein rarely exceeds 100mg%, and while modest elevations ofCSF WBC count are common, thenumber rarely exceeds 50/cu.mm.

Page 134: usmle Board Review Step 1

134www.brain101.info

A 21 year old woman is brought to the emergency room because ofintense vertigo. She had been in good health until 2 days prior whenshe first experienced nausea and mild vertigo. The next day she noteddiplopia. Past medical history is significant for a bout of optic neuritison the right lasting 2 weeks when she was age 18. The optic neuritiscompletely resolved over 3 weeks. You are called to the emergencyroom to consult on the patients current condition. Your examinationdemonstrates paleness of the left optic nerve, bilateral internuclearophthalmopelegia and severe vertigo precipitated immediately by smallmovements of the head. The patient has intention tremors in bothupper extremities. Motor strength is normal in the upper extremities;however, she has mild weakness in both lower extremities. Reflexesare normal in the upper extremities and hyperactive in the lowerextremities. Babinski signs are present bilaterally. Sensoryexamination reveals a level of numbness from the T12 dermatomes ondown to the feet bilaterally.

The clinical examination demonstrates abnormalities in :A. Optic nerveB. Brainstem and CerebellumC. Thoracic spinal cordD. All of the aboveE. A & C only

Page 135: usmle Board Review Step 1

135www.brain101.info

The correct answer is D.This condition affects multiple areas ofwhite matter within the central nervoussystem.

Page 136: usmle Board Review Step 1

136www.brain101.info

A 21 year old woman is brought to the emergency room because ofintense vertigo. She had been in good health until 2 days prior whenshe first experienced nausea and mild vertigo. The next day she noteddiplopia. Past medical history is significant for a bout of optic neuritison the right lasting 2 weeks when she was age 18. The optic neuritiscompletely resolved over 3 weeks. You are called to the emergencyroom to consult on the patients current condition. Your examinationdemonstrates paleness of the left optic nerve, bilateral internuclearophthalmopelegia and severe vertigo precipitated immediately by smallmovements of the head. The patient has intention tremors in bothupper extremities. Motor strength is normal in the upper extremities;however, she has mild weakness in both lower extremities. Reflexesare normal in the upper extremities and hyperactive in the lowerextremities. Babinski signs are present bilaterally. Sensoryexamination reveals a level of numbness from the T12 dermatomes ondown to the feet bilaterally.

Which test(s) or evaluations would you proceed with?A. MRI scans of the brain and/or spinal cordB. Neurosurgical consultationC. Spinal tap to determine if oligoclonal bands are presentD. All of the aboveE. A & C only

Page 137: usmle Board Review Step 1

137www.brain101.info

The correct answer is E.Radiological studies are often useful to be certain thatother causes of neurologic disease are not present aswell as to confirm the presence of lesions in the whitematter. MRI scans are more useful and may showcharacteristic white matter lesions in over 80% ofpatients. Spinal taps are helpful because CSFgammaglobulin is elevated in ~75% of patients at sometime during the disease. In addition, when agarose gel,acrylamide electrophoresis or isoelectric focusing areused, it can be observed that gammaglobulin migratesin more than one band (oligoclonal pattern) in 80-95%of patients even if their gammaglobulin is normal.

Page 138: usmle Board Review Step 1

138www.brain101.info

A 21 year old woman is brought to the emergency room because ofintense vertigo. She had been in good health until 2 days prior whenshe first experienced nausea and mild vertigo. The next day she noteddiplopia. Past medical history is significant for a bout of optic neuritison the right lasting 2 weeks when she was age 18. The optic neuritiscompletely resolved over 3 weeks. You are called to the emergencyroom to consult on the patients current condition. Your examinationdemonstrates paleness of the left optic nerve, bilateral internuclearophthalmopelegia and severe vertigo precipitated immediately by smallmovements of the head. The patient has intention tremors in bothupper extremities. Motor strength is normal in the upper extremities;however, she has mild weakness in both lower extremities. Reflexesare normal in the upper extremities and hyperactive in the lowerextremities. Babinski signs are present bilaterally. Sensoryexamination reveals a level of numbness from the T12 dermatomes ondown to the feet bilaterally.

The most likely diagnosis is:A. Herpes encephalitisB. Multiple sclerosisC. Brainstem strokeD. Cerebellar hemorrhageE. Ruptured basilar artery aneurysm

Page 139: usmle Board Review Step 1

139www.brain101.info

The correct answer is B.MS is rare before puberty and ordinarliy starts beforeage 50. Symptoms include motor weakness,paresthesia and other sensory disorders, optic neuritis,unsteady gait and other cerebellar signs. Symptomsmay develop rapidly in the course of hours or days orless frequently may take weeks to reach their maximum.The Babinski response consists of dorsiflexion of thebig toe and fanning of other toes in response to strokingthe lateral border of the foot (S1 dermatome); flexion atthe hip and knee may also occur. Such a responseindicates an upper motor neuron lesion involving thecontralateral motor cortex or the corticospinal tract.

Page 140: usmle Board Review Step 1

140www.brain101.info

In narcolepsy the following are usually foundA. Polysomnography is abnormal and themultiple sleep latency test is normalB. HLA type DR2/DQW1C. The full tetrad of daytime sleepiness, sleepparalysis, hypnagogic hallucinations andcataplexyD. All of the aboveE. A&C only

Page 141: usmle Board Review Step 1

141www.brain101.info

The correct answer is B.HLA markers DR2/DQW1 are found muchmore frequently in narcoleptics than in thegeneral population.

Page 142: usmle Board Review Step 1

142www.brain101.info

Restless Legs Syndrome is associatedwithA. UremiaB. Iron deficiencyC. Insomnia in the elderlyD. All of the aboveE. A&C only

Page 143: usmle Board Review Step 1

143www.brain101.info

The correct answer is D.This is a neurologic cause of insomnia ordaytime sleepiness in the elderly, theuremic, and those with a positive familyhistory. It is also associated with irondeficiency, peripheral neuropathy, andperipheral vascular disease.

Page 144: usmle Board Review Step 1

144www.brain101.info

The normal sleep architecture featuresslow wave sleep (stages 3&4) in themiddle third of the night.A. TrueB. False

Page 145: usmle Board Review Step 1

145www.brain101.info

B. The correct answer is False.SWS is concentrated during the first thirdof the night. (REM is concentrated duringthe last third of the night. About half of thenight is spent in stage 2 sleep.)

Page 146: usmle Board Review Step 1

146www.brain101.info

In Locked In Syndrome the patient isunconscious.A. TrueB. False

Page 147: usmle Board Review Step 1

147www.brain101.info

B. The correct answer is False.Locked in syndrome ("coma vigile") is nota form of coma-consciousness ispreserved, but patient's motor function issufficiently impaired to prevent outwardexpression of thought and behavior. (Dueto extensive or transverse high brainstemlesions.)

Page 148: usmle Board Review Step 1

148www.brain101.info

Swelling of the hemispheres and basalganglia bilaterally with compression of thediencephalon and adjoining midbrain ischaracteristic of:A. Central HerniationB. UncalHerniationC. Cingulate Gyrus HerniationD. All of the aboveE. None of the above

Page 149: usmle Board Review Step 1

149www.brain101.info

The correct answer is A.Central herniation: Involves swelling ofhemispheres and basal ganglia bilaterally(usually), Compresses diencephalon andadjoining midbrain caudally through the tentorialnotch, and Results in diencephalic and midbrain(upper brainstem) damage secondary tocompression, ischemia and or hemorrhage(traction on penetrating vessels of midbrain andpons.)

Page 150: usmle Board Review Step 1

150www.brain101.info

Pinpoint pupils in the unconscious patientare consistent with which level ofinvolvement:A. DiencephalonB. TectumC. MidbrainD. PonsE. None of the above

Page 151: usmle Board Review Step 1

151www.brain101.info

The correct answer is D.Note: In addition to this and the above,another level which can be identified bypupil involvement is compression of thethird cranial nerve in uncal herniation.This will cause a unilateral, fixed, dilatedpupil ("blown pupil")

Page 152: usmle Board Review Step 1

152www.brain101.info

Decerebrate posturing is seen inunconscious patients with lesions orcompression at which level.A. Midbrain or PonsB. Diencephalon

Page 153: usmle Board Review Step 1

153www.brain101.info

The correct answer is A.Lower lesions (midbrain-pons) causedecerebrate posturing.

Page 154: usmle Board Review Step 1

154www.brain101.info

Initial evaluation of the spinal cord injuredpatient requires:A. Careful attention to the "ABCs" ofresuscitation (airway, breathing andcirculation)B. Emergency MRI scanC. EMG (electromyogram)D. HyperventilationE. All of the above

Page 155: usmle Board Review Step 1

155www.brain101.info

The correct answer is A.Initial evaluation involves careful attentionto “ABCs” of resuscitation and alsocareful attention to associated injuries. Inaddition, the spinal column needs to beimmobilized.

Page 156: usmle Board Review Step 1

156www.brain101.info

Matching the reflex with its neurologiclevel:Ankle jerkA. C5-C6B. L3-L4C. S1-S2D. C7-C8

Page 157: usmle Board Review Step 1

157www.brain101.info

The correct answer is C.(Tibial nerve)

Page 158: usmle Board Review Step 1

158www.brain101.info

Matching the reflex with its neurologiclevel:Knee jerkA. C5-C6B. L3-L4C. S1-S2D. C7-C8

Page 159: usmle Board Review Step 1

159www.brain101.info

The correct answer is B.(Femoral nerve)

Page 160: usmle Board Review Step 1

160www.brain101.info

Matching the reflex with its neurologiclevel:Biceps jerkA. C5-C6B. L3-L4C. S1-S2D. C7-C8

Page 161: usmle Board Review Step 1

161www.brain101.info

The correct answer is A.(Musculocutaneous nerve)

Page 162: usmle Board Review Step 1

162www.brain101.info

Which is/are true of Parkinson Disease:A. The disease is characterized bybradykinesia, tremor and rigidityB. There is prominent loss of neurons in thesubstantia nigraC. Lack of facial expression (Masked facies)and diminshed blink rate are common featuresD. All of the aboveE. A & C only

Page 163: usmle Board Review Step 1

163www.brain101.info

The correct answer is D.Onset is with tremor. Patient simultaneouslydevelops rigidity and brady kinesia which ismanifested by a stooped, fixed posture, maskedfacies, diminished blink rate, difficulty initiatingand maintaing movements, propulsion andretropulsion, and typical festinating gait.Pathology consists of degeneration ofpigmented neurons of the mesencephalon andbrain stem. Most prominent is the loss ofdopamine cell bodies in the substantia nigra.

Page 164: usmle Board Review Step 1

164www.brain101.info

Tourette's syndrome may be transmittedas an autosomal recessive trait withstriking uniformity of phenotypicexpression.A. TrueB. False

Page 165: usmle Board Review Step 1

165www.brain101.info

B. The correct answer is False.Tourette’s Syndrome may be transmittedas an autosomal dominant trait with atremendous variability in phenotypicexpression.

Page 166: usmle Board Review Step 1

166www.brain101.info

The genetics of Huntington's chorea ischaracterized by which of the following:A. Expansion of a trinucleotide repeatB. The gene is located on the shortarm of chromosome 4C. Autosomal dominant inheritanceD. All of the aboveE. A&C only

Page 167: usmle Board Review Step 1

167www.brain101.info

The correct answer is D.It is characterized by the triad ofautosomal dominant inheritance,movement disorder, & dementia. Thegene is located on the terminal band ofthe short arm of chromosome 4. Themutation results from the expansion of aCAG repeat.

Page 168: usmle Board Review Step 1

168www.brain101.info

Clinical manifestations of Huntington'schorea include all with the exception of:A. Emotional lability and depressionB. DementiaC. Rigidity and spasticity in the laterstages of the diseaseD. Profound sensory loss

Page 169: usmle Board Review Step 1

169www.brain101.info

The correct answer is D.Profound sensory loss is not a symptomof Huntington's

Page 170: usmle Board Review Step 1

170www.brain101.info

Management of elevated intracranial pressureincludes:A. Modest hyperventilation to pCO2 of 30 to 35B. MannitolC. Elevation of the head 20 to 30 degreesavoiding neck vein compressionD. Avoidance of antihypertensive medicationsE. All of the above

Page 171: usmle Board Review Step 1

171www.brain101.info

The correct answer is E.Intubate with modest hyperventilation topCO2 of 30, IV Mannitol, IV Furosemide,Elevate head 20 - 30 degrees avoidingneck vein compression, sedate with IVmorphine as needed to keep quiet orparalyze with pancuronium, avoidhypertensives.

Page 172: usmle Board Review Step 1

172www.brain101.info

Major complications of SubarchnoidHemorrhage Include:A. RebleedB. HydrocephalusC. VasospasmD. All of the aboveE. A & C only

Page 173: usmle Board Review Step 1

173www.brain101.info

The correct answer is D.Major complications include: Rebleed,Vasospasm (symptomatic ischemia orinfarct), Hydrocephalus, Cardiac(subendocardial infarction, arrhythmias),and Pulmonary (neurogenic edema)

Page 174: usmle Board Review Step 1

174www.brain101.info

Torn bridging veins and/or arteriolesA. Subarachnoid hemorrhageB. Epidural hemorrhageC. Acute subdural hemorrhageD. Intracerebral hematoma

Page 175: usmle Board Review Step 1

175www.brain101.info

The correct answer is C.Usually traumatic, from torn brainarterioles or bridging vein. Usuallyassociated with severe brain injury,hemispheric contusions or diffuse shockinjury.

Page 176: usmle Board Review Step 1

176www.brain101.info

Hemorrhage in the basal gangliaA. Subarachnoid hemorrhageB. Epidural hemorrhageC. Acute subdural hemorrhageD. Intracerebral hematoma

Page 177: usmle Board Review Step 1

177www.brain101.info

The correct answer is D.Sites of hematomas include basalganglia/internal capsule (causeshemiparesis and dysphasia), cerebellar(causes ataxia and dizziness) and pontine(causes coma and cranial nerve deficits.)

Page 178: usmle Board Review Step 1

178www.brain101.info

Torn middle meningeal arteryA. Subarachnoid hemorrhageB. Epidural hemorrhageC. Acute subdural hemorrhageD. Intracerebral hematoma

Page 179: usmle Board Review Step 1

179www.brain101.info

The correct answer is B.Traumatic, usually torn meningeal artery.Commonly associated with skull fracture.Brain is usually without significant injury.

Page 180: usmle Board Review Step 1

180www.brain101.info

Which is/are symptomatic of a transientischemic attack (TIA).A. SyncopeB. Transient monocular blindnessC. Vertigo by itselfD. All of the aboveE. A & C only

Page 181: usmle Board Review Step 1

181www.brain101.info

The correct answer is B.Numbness of hand and cheek, weaknessof hand or leg, transient monocularblindness, and less commonly languagedisturbance, are symptoms of TIA.

Page 182: usmle Board Review Step 1

182www.brain101.info

Cardiac sources of emboli includeA. Prosthetic heart valvesB. Mitral stenosisC. Atrial fibrillationD. Mural (heart wall) thrombus followinganterior myocardial infarctionE. All of the above

Page 183: usmle Board Review Step 1

183www.brain101.info

The correct answer is E.Cardiac source for embolism includes:Atrial fibrillation, Mitral stenosis, Aorticstenosis, Prosthetic heart valves,Ventricular aneurysm, Following anteriorMI, Dilated cardiomyopathy, Paradoxicalthrough atrial septal defect, congenitalheart disease, Bacterial endocarditis, andMarantic endocarditis.

Page 184: usmle Board Review Step 1

184www.brain101.info

Matching Stroke Manifestations withNeuroanatomic localizationPure motor strokeA. ThalamusB. Internal capsuleC. Dominant frontal lobe

Page 185: usmle Board Review Step 1

185www.brain101.info

The correct answer is B.Pure motor stroke implicates an insultwhere corticospinal fibers are closelygrouped, such as the internal capsule.

Page 186: usmle Board Review Step 1

186www.brain101.info

Matching Stroke Manifestations withNeuroanatomic localizationExpressive aphasia with right arm andfacial weaknessA. ThalamusB. Internal CapsuleC. Dominant Frontal Lobe

Page 187: usmle Board Review Step 1

187www.brain101.info

The correct answer is C.The presence of aphasia and thedifference in weakness between face/arm& leg localizes the injury to dominantfrontal lobe cortex & white matter.

Page 188: usmle Board Review Step 1

188www.brain101.info

Matching Stroke Manifestations withNeuroanatomic localizationPure sensory lossA. ThalamusB. Internal CapsuleC. Dominant Frontal Lobe

Page 189: usmle Board Review Step 1

189www.brain101.info

The correct answer is A.This implicates the thalamus and is alsousually lacunar.

Page 190: usmle Board Review Step 1

190www.brain101.info

The spinal fluid in viral meningitis istypically characterized by lymphocytesand normal glucose:

A. True B. False

Page 191: usmle Board Review Step 1

191www.brain101.info

A. The correct answer is True.Although there is a transientpredominance of PMNs within the firstfew hours, the CSF formula quicklychanges to a predominant lymphocytosisaccompanied by a slight rise in protein.Glucose values are usually in the normalrange.

Page 192: usmle Board Review Step 1

192www.brain101.info

Which is the most frequent non-epidemicnecrotizing encephalitis in the UnitedStates?A. Western EquineB. CaliforniaC. St. LouisD. Eastern EquineE. Herpes

Page 193: usmle Board Review Step 1

193www.brain101.info

The correct answer is E.Herpes is non-epidemic.

Page 194: usmle Board Review Step 1

194www.brain101.info

Which are true for Herpes encepahlitis:A. If the illness is suspected then acyclovirshould be started immediately and then an MRIand spinal fluid examination should beobtained.B. The EEG is rarely abnormalC. The PCR on spinal fluid is very infrequentlypositive for Herpes DNAD. All of the aboveE. A&C only

Page 195: usmle Board Review Step 1

195www.brain101.info

The correct answer is A.Acyclovir reduces the mortality of Herpesencephlitis, but there is still a significantmorbidity among survivors so it isimperative to establish the diagnosis ofthis condition at the earliest possible time.

Page 196: usmle Board Review Step 1

196www.brain101.info

Basilar meningitis may have severeconsequences which include:A. VasculitisB. HydrocephalusC. Cranial nerve VII palsyD. All of the aboveE. A & C only

Page 197: usmle Board Review Step 1

197www.brain101.info

The correct answer is D.Other severe consequences includecortical ischemia due to vasculitis w/occlusion or loss of vascular autoregulation, and increased intracranialpressure.

Page 198: usmle Board Review Step 1

198www.brain101.info

The most frequent causes of bacterialmeningitis in the adult are:A. Pneumococcus and meningococcusB. Hemophilus influenzae andmeningococcusC. Staph aureus and listeriamonocytogenesD. Staph aureus and E. coli

Page 199: usmle Board Review Step 1

199www.brain101.info

The correct answer is A.

Page 200: usmle Board Review Step 1

200www.brain101.info

The spinal fluid in bacterial meningitis ischaracterized by:A. Polymorphonuclear leukocytes andincreased glucoseB. Increased protein and increased glucoseC. Polymorphonuclear leukocytes anddecreased glucoseD. Decreased protein and increased glucose

Page 201: usmle Board Review Step 1

201www.brain101.info

The correct answer is C.In addition, protein is increased and gramstain detection of bacteria is oftenpositive.

Page 202: usmle Board Review Step 1

202www.brain101.info

The spinal fluid in bacterial meningitis ischaracterized by:A. Polymorphonuclear leukocytes andincreased glucoseB. Increased protein and increased glucoseC. Polymorphonuclear leukocytes anddecreased glucoseD. Decreased protein and increased glucose

Page 203: usmle Board Review Step 1

203www.brain101.info

The correct answer is C.

You are correct! In addition, protein isincreased and gram stain detection ofbacteria is often positive.

Page 204: usmle Board Review Step 1

204www.brain101.info

A 29 year old HIV positive man with previous and ongoing intravenousdrug use is initially referred to you for evaluation of pain and numbness inthe feet. Examination, at that time, revealed impaired sensation to pin,temperature and touch to the mid calf level symmetrically and absentankle reflexes. The patient did not have Babinski signs and strength isnormal in all extremities. Nerve conduction studies demonstrate milddecrease in nerve conduction velocities in the all four extremities.Two months later you see the patient in the clinic because of a newcomplaint of increasing neck pain for the past three days. Pain is nowradiating from the neck into the upper extremities and the patient iscomplaining of weakness in the legs. The patient has a fever of 101.5degrees and pain on motion of the neck in all directions. Your neurologicexamination shows depressed biceps and increased triceps reflexes.Knee jerks (patellar reflexes) are increased and the patient now hasBabinski signs bilaterally. You also find mild weakness and spasticity inboth legs.The patients second problem (neck pain with radiation to the arms) willrequire evaluationWhich test should be obtained?A. MRI of the cervical spineB. Nerve biopsyC. Repeat nerve conduction studiesD. Lumbar puncture (spinal tap) to determine CSF protein and cell counts

Page 205: usmle Board Review Step 1

205www.brain101.info

The correct answer is A.MRI is the diagnostic procedure of choicefor focal infections, followed by CT scan.

Page 206: usmle Board Review Step 1

206www.brain101.info

A 29 year old HIV positive man with previous and ongoing intravenousdrug use is initially referred to you for evaluation of pain and numbnessin the feet. Examination, at that time, revealed impaired sensation topin, temperature and touch to the mid calf level symmetrically andabsent ankle reflexes. The patient did not have Babinski signs andstrength is normal in all extremities. Nerve conduction studiesdemonstrate mild decrease in nerve conduction velocities in the all fourextremities.Two months later you see the patient in the clinic because of a newcomplaint of increasing neck pain for the past three days. Pain is nowradiating from the neck into the upper extremities and the patient iscomplaining of weakness in the legs. The patient has a fever of 101.5degrees and pain on motion of the neck in all directions. Yourneurologic examination shows depressed biceps and increased tricepsreflexes. Knee jerks (patellar reflexes) are increased and the patientnow has Babinski signs bilaterally. You also find mild weakness andspasticity in both legs.The most likely diagnosis is:A. Chronic inflammatory demyelinating polyneuropathyB. HIV myelopathyC. Spinal epidural abscessD. Ruptured cervical diskE. Spinal meningioma

Page 207: usmle Board Review Step 1

207www.brain101.info

The correct answer is C.Symptoms of spinal epidural abscessoccur in 4 stages: Focal pain, Radicularpain, Long tract signs, and Signs of cordtransection.

Page 208: usmle Board Review Step 1

208www.brain101.info

Match the feature(s) with the myopathy:Cardiac Conduction DefectA. PolymyositisB. Myotonic DystrophyC. Duchenne's DystrophyD. All of the Above

Page 209: usmle Board Review Step 1

209www.brain101.info

The correct answer is B.Other associated features include: Frontalbalding, Cataracts, Intellectual dullness(esp. younger onset), Abdominal pain,Intestinal dysmotility, Testicular atrophy,Endocrine & bone abnormalities andRespiratory insufficiency.

Page 210: usmle Board Review Step 1

210www.brain101.info

Match the feature(s) with the myopathy:Enlarged Trinucleotide RepeatA. PolymyositisB. Myotonic DystrophyC. Duchenne's DystrophyD. All of the Above

Page 211: usmle Board Review Step 1

211www.brain101.info

The correct answer is B.Myotonic Dystrophy is due to atrinucleotide repeat in the myotoninprotein knase (MPK) gene onchromosome 19. The trinucleotide repeatincreases in size in successive generationcausing anticipation.

Page 212: usmle Board Review Step 1

212www.brain101.info

Match the feature(s) with the myopathy:Autosomal dominant inheritanceA. PolymyositisB. Myotonic DystrophyC. Duchenne's DystrophyD. All of the Above

Page 213: usmle Board Review Step 1

213www.brain101.info

The correct answer is B.

Page 214: usmle Board Review Step 1

214www.brain101.info

Match the feature(s) with the myopathy:Gower's sign and X-linked RecessiveInheritanceA. PolymyositisB. Myotonic DystrophyC. Duchenne's DystrophyD. All of the Above

Page 215: usmle Board Review Step 1

215www.brain101.info

The correct answer is C.Gower’s sign: Patient places hand orhands on knees and effectively “crawls upthe legs” to arise from the floor. Otherfindings include: Pelvic & shoulder girdleweakness, Calf hypertrophy, Waddlinggait, Lordosis, Abnormal (or inability to)run, and Heel cord contractures.

Page 216: usmle Board Review Step 1

216www.brain101.info

Match the visual field with the location:Bitemporal hemianopiaA. Right Optic NerveB. Right Parietal LobeC. Right Temporal LobeD. Left Occipital LobeE. Optic Chiasm

Page 217: usmle Board Review Step 1

217www.brain101.info

The correct answer is E.The major lesions that produce visualimpairment at the level of the opticchiasm are tumors, especially of pituitaryorigin. The classic pattern of visual deficitis bitemporal hemianopia.

Page 218: usmle Board Review Step 1

218www.brain101.info

Match the visual field with the location:Left superior quadrantanopiaA. Right Optic NerveB. Right Parietal LobeC. Right Temporal LobeD. Left Occipital LobeE. Optic Chiasm

Page 219: usmle Board Review Step 1

219www.brain101.info

The correct answer is C.With lesions in the temporal lobe, wheretumors are the most common cause, thefield deficit is denser superiorly thaninferiorly, resulting in a superiorquandrantanopia (pie in the sky deficit).

Page 220: usmle Board Review Step 1

220www.brain101.info

Match the visual field with the location:Blind in Right eyeA. Right Optic NerveB. Right Parietal LobeC. Right Temporal LobeD. Left Occipital LobeE. Optic Chiasm

Page 221: usmle Board Review Step 1

221www.brain101.info

The correct answer is A.A complete lesion of the right optic nervecauses blindness in the right eye.

Page 222: usmle Board Review Step 1

222www.brain101.info

A stillbirth occurs at 28 weeks gestation to a 30 year oldG2 P1 woman whose previous pregnancy resulted in anormal term birth. At autopsy, the cerebrum of the fetusdemonstrates extensive diffuse periventricular areas ofnecrosis. Which of the following infections is most likelyto have caused these findings:A Taenia soliumB CytomegalovirusC PoliovirusD CandidaE Syphilis

Page 223: usmle Board Review Step 1

223www.brain101.info

(B) CORRECT. Remember TORCH forcongenital infections. The 'T' fortoxoplasmosis and the 'C' forcytomegalovirus are most likely to involvethe CNS. The extent of necrosis andcalcification with cytomegalovirus can beconsiderable.

Page 224: usmle Board Review Step 1

224www.brain101.info

A 73 year old male has exhibited problemsremembering things for several months, and he is notedto confabulate. He dies as a consequence of ahepatocellular carcinoma. At autopsy, his braindemonstrates bilaterally small mammillary bodies thatshow brown discoloration. Microscopically, there isgliosis and vascular proliferation and hemosiderindeposition. These findings are most typical for:A Multiple sclerosisB Parkinson's diseaseC Amyotrophic lateral sclerosisD Wernicke-Korsakoff syndromeE Huntington's disease

Page 225: usmle Board Review Step 1

225www.brain101.info

(D) CORRECT. Wernicke's disease canalso lead to hemorrhage and/or loss ofperiaqueductal grey matter. TheWernicke-Korsakoff syndrome is seenwith chronic alcoholism. The mechanismmay have to do with thiamine deficiency.(Micronodular cirrhosis with alcoholism isa risk for hepatocellular carcinoma.)

Page 226: usmle Board Review Step 1

226www.brain101.info

A 50-year-old male complained of headaches,becoming irritable and acting strangely about a monthafter being involved in a vehicular accident in which hewas not wearing any restraint and struck his headagainst the windshield of his van. He did not loseconsciousness at that time or at any point therafter. Hehad a minor contusion to his forhead. This history ismost consistent with the development of a (an):A Epidural hematomaB Chronic subdural hematomaC Cerebral contusionsD Subarachnoid hemorrhageE Intracerebral hematoma

Page 227: usmle Board Review Step 1

227www.brain101.info

(B) CORRECT. The continued presenceof a subdural hematoma leads to theproblems described, even if it is relativelysmall.

Page 228: usmle Board Review Step 1

228www.brain101.info

A young healthy male, a major league baseball player,developed progressive, symmetric muscular weaknessof his upper extremities over the course of severalyears. Then he developed difficulty speaking andswallowing. He did not have myalgias or arthralgias. Heremained afebrile. His mental function never becamediminished. He is most likely to have:A Amyotrophic lateral sclerosisB von Recklinghausen's diseaseC Multiple sclerosisD Werdnig-Hoffman diseaseE Guillain-Barre syndrome

Page 229: usmle Board Review Step 1

229www.brain101.info

(A) CORRECT. The course isprogressive. Bulbar involvement can leadto problems speaking and eating, with riskfor aspiration. This disease often goes bythe eponym of another baseball player,Lou Gehrig, who could not play first baseanymore.

Page 230: usmle Board Review Step 1

230www.brain101.info

A 20 year old previously healthy male has recently been inductedinto the army. Several weeks into basic training, he experiences asevere headache for an entire day. He had been healthy prior tothis, noting only a mild sore throat the prior day. He goes to thebase physician, who records vital signs of T 39.2 C, P 90, R 22,and BP 110/70 mm Hg. A lumbar puncture is performed andexamination of the cerebrospinal fluid shows 2 RBC's, 34,000WBC's, glucose of 20 mg/dl (serum 75 mg/dL), and protein of 105mg/dl. Which of the following additional laboratory tests would bethe most helpful in reaching a diagnosis:A Cryptococcal antigenB Acid fast stainC India inkD Serology for Herpes simplexE Gram stain

Page 231: usmle Board Review Step 1

231www.brain101.info

(E) CORRECT. The findings point to abacterial infection. The most likelyorganism at this age and under thesecircumstances is Neisseria meningitidis.The portal of infection may be apharyngitis.

Page 232: usmle Board Review Step 1

232www.brain101.info

At autopsy, the brain of a 47 year old male is normal in size, withno cortical atrophy. The cerebral arteries show no atherosclerosis.Coronal sections reveal scattered periventricular plaques ofdemyelination from 0.3 to 1 cm in size. Microscopically, theseplaques have loss of myelin as seen with the luxol fast blue (LFB)stain, but axonal preservation as seen with the Bodian silver stain.This disease process is characterized by all of the followingfindings EXCEPT:A Onset in early adulthoodB Oligoclonal bands in the CSFC Association with influenza virus infectionD Plaques of demyelination in white matterE A relapsing and remitting course

Page 233: usmle Board Review Step 1

233www.brain101.info

(C) CORRECT. This is the falsestatement. Multiple sclerosis (MS) is notassociated with influenza virus.

Page 234: usmle Board Review Step 1

234www.brain101.info

A 45 year old man has had a severe headache for a week.Physical examination reveals papilledema on the right. A head CTscan reveals a marked right to left midline shift. An MRI scandemonstrates a 6 cm enhancing mass lesion in the right parietalregion with marked surrounding edema. He develops a dilatedpupil on the right and soon thereafter loses consciousness anddies. At autopsy, which of the following lesions is most likely to befound:A Superior sagittal sinus thrombosisB Right cerebellar hemispheric hemorrhageC Pontine hemorrhagesD Thrombosis of the posterior cerebral arteryE Diffuse subarachnoid hemorrhage

Page 235: usmle Board Review Step 1

235www.brain101.info

(C) CORRECT. The brain swelling leadsto herniation, which damages smallperforating vessels and results in Durethemorrhages in the pons and midbrain.

Page 236: usmle Board Review Step 1

236www.brain101.info

A 43 year old woman develops a progressive, ascending motorweakness over several days. She is hospitalized and requiresintubation with mechanical ventilation. She is afebrile. A lumbarpuncture is performed with normal opening pressure and yieldsclear, colorless CSF with normal glucose, increased protein, andcell count of 5/microliter, all lymphocytes. She gradually recoversover the next month. Which of the following conditions most likelypreceded the onset of her illness:A KetoacidosisB Viral pneumoniaC Staphylococcus aureus septicemiaD Systemic lupus erythematosusE Vitamin B12 deficiency

Page 237: usmle Board Review Step 1

237www.brain101.info

(B) CORRECT. Guillain-Barré syndrome,an acute idiopathic polyneuritis, is felt tobe immunologic. About 75% of caseshave a history of a preceding infection,including viral (cytomegalovirus) andbacterial (Campylobacter jejuni) agents.

Page 238: usmle Board Review Step 1

238www.brain101.info

A 50 year old female has become comatose.She has papilledema on the right onfunduscopic examination. Cerebellar tonsillarherniation is suspected. Which of the followingconditions would be LEAST likely to explainthese findings in this patient:A Wernicke's diseaseB Cerebral abscessC Glioblastoma multiformeD Chronic subdural hematomaE Hypertensive intraparenchymal hemorrhage

Page 239: usmle Board Review Step 1

239www.brain101.info

(A) CORRECT. The lesions of Wernicke'sdisease are unlikely to increaseintracranial pressure.

Page 240: usmle Board Review Step 1

240www.brain101.info

The mother of a 5 year old girl realizes that her childhas spent all of Saturday in bed. The girl is listless andnot arousable, so her mother takes the child to theemergency room. The examining physician notes atemperature of 38.8 C and nuchal rigidity. A lumbarpuncture yields slightly cloudy CSF with an elevatedprotein and decreased glucose. A culture of CSF ismost likely to yield:A Hemophilus infuenzaeB Cryptococcus neoformansC Aspergillus fumigatusD Mycobacterium tuberculosisE Cytomegalovirus

Page 241: usmle Board Review Step 1

241www.brain101.info

(A) CORRECT. The findings point to abacterial meningitis, and H. influenzae isthe most likely organism to cause thisfinding in children.

Page 242: usmle Board Review Step 1

242www.brain101.info

A 45 year old female noticed tinnitus in her left ear whichprogressed over some weeks to hearing loss in that ear. Onphysical examination she is found to have a marked decrease inhearing on the left, with Rinne test indicating air conduction betterthan bone conduction. The other cranial nerves I - VII and IX - XIIare intact. A brain MRI scan revealed a solitary, fairly discreet, 3cm mass located in the region of the left cerebellopontine angle.Which of the following statements is most appropriate to tell thepatient regarding these findings:A You are unlikely to survive for more than a yearB Remissions and exacerbations are likely to occur in comingyearsC Other family members should undergo MR imaging of the brainD The lesion can be resected with a good prognosisE A test for HIV-1 is likely to be positive

Page 243: usmle Board Review Step 1

243www.brain101.info

(D) CORRECT. The findings arecharacteristic for schwannoma, called anacoustic neuroma when the 8th cranialnerve is involved. These are benignneoplasms. A solitary mass is unlikely tobe part of neurofibromatosis.

Page 244: usmle Board Review Step 1

244www.brain101.info

A 65 year old male has general paresis with increasingloss of higher mental functions. A VDRL is positive oncerebrospinal fluid obtained by lumbar puncture. TheCSF protein and glucose are normal, and there is 1mononuclear cell present. Which of the followingpathologic findings is UNLIKELY to be a feature of hisdisease:A Cortical neuronal loss with atrophyB Hemorrhagic encephalitisC Chronic meningitisD EndarteritisE Atrophy of spinal cord dorsal columns

Page 245: usmle Board Review Step 1

245www.brain101.info

(B) CORRECT. This is not a feature ofneurosyphilis. Hemorrhagic encephalitiswould be more typical of herpes simplexvirus infection

Page 246: usmle Board Review Step 1

246www.brain101.info

A 53 year old woman has had transient ischemicattacks (TIAs) for several years. She then has thesudden onset of a left hemiparesis. Four months later,an MRI scan of the brain shows findings consistent witha cystic 4 cm area in the right frontal-parietal region.Which of the following underlying conditions is she mostlikely to have:A Occlusive coronary atherosclerosisB Chronic meningitisC Alzheimer diseaseD Glioblastoma multiformeE Cerebral arterial vasculitis

Page 247: usmle Board Review Step 1

247www.brain101.info

(A) CORRECT. These findings suggest a'stroke' from cerebral infarction. Mostbrain infarcts result fromthromboembolism. The most commonsource for emboli is the heart. Coronaryatherosclerosis can result in myocardialinfarction with overlying endocardial muralthrombosis. Such mural thrombi canembolize to the systemic circulation.

Page 248: usmle Board Review Step 1

248www.brain101.info

A 50-year-old male has a history of falling multiple timesover the past few years. On his last hospital admission,he was noted to have a contusion on the scalp at hisocciput. His blood ethanol was .29 gm% (290 mg/dL).Which of the following lesions is LEAST likely to befound in this patient:A Cerebral contusionsB Basal ganglia hemorrhageC Wernicke's diseaseD Subdural hematomaE Central pontine myelinolysis

Page 249: usmle Board Review Step 1

249www.brain101.info

(B) CORRECT. Such hemorrhages are afeature of hypertension, not alcoholism.

Page 250: usmle Board Review Step 1

250www.brain101.info

A 54 year old female develops a a distal, symmetric,primarily sensory polyneuropathy over a period ofseveral months. She also has a non-healing ulcerationon the ball of her left foot. She had a myocardialinfarction last year but recovered and is doing wellfollowing angioplasty. Which of the following laboratorytest findings would you most likely expect to be present:A Decreased (20 mg/dL) glucose in CSFB Pap smear showing changes of herpes simplex virusinfectionC Markedly increased blood lead of 50 micrograms/dLD Chromosome analysis with a 47, XX, +21 karyotypeE Elevated serum glucose of 195 mg/dL

Page 251: usmle Board Review Step 1

251www.brain101.info

(E) CORRECT. Diabetic neuropathy isprobably the most common form ofperipheral neuropathy in the UnitedStates and Europe. She also has a'diabetic foot' from severe peripheralvascular atherosclerosis, and the MI isconsistent with severe occlusive coronaryatherosclerosis.

Page 252: usmle Board Review Step 1

252www.brain101.info

A 53-year-old male has a 6 month course of rapidlyprogressive dementia along with myoclonus. A cerebralelectroencephalogram shows periodic biphasicsynchronous sharp-wave complexes that aresuperimposed upon a slow background rhythm. He diesfrom bronchopneumonia. At autopsy, his brain appearsgrossly normal, but a spongiform encephalopathy isseen microscopically in a section of the cerebral cortex(which was put in concentrated formic acid for 1 hourprior to processing). He most likely has:A Alzheimer's diseaseB Creutzfeldt-Jakob diseaseC AIDS dementiaD RabiesE Herpes viral encephalitis

Page 253: usmle Board Review Step 1

253www.brain101.info

(B) CORRECT. The incidence of thisdisease is about 1 per 1,000,000. It iscaused by an abnormal prion protein. Theformic acid pretreatment of the specimenwill inactivate this protein.

Page 254: usmle Board Review Step 1

254www.brain101.info

An infant displays failure to reach developmentalmilestones. There is a prominent 2 cm lumbarmeningomyelocele. An MRI scan of the braindemonstrates downward extension of the cerebellarvermis and displacement of the medulla from a smallposterior fossa into the foramen magnum. There istenting of the tectum of the midbrain. The cerebralventricles are enlarged. The spinal cord has findings ofhydromyelia. Which of the following conditions is mostlikely to account for these findings:A Dandy-Walker malformationB Viral meningoencephalitisC Arnold-Chiari malformationD Maternal folate deficiencyE Werdnig-Hoffman disease

Page 255: usmle Board Review Step 1

255www.brain101.info

(C) CORRECT. The findings are those ofa Chiari type II malformation.

Page 256: usmle Board Review Step 1

256www.brain101.info

A 22 year old male has recently emigrated from MexicoCity. He has the sudden onset of a seizure disorderwhile working as a chef in a restaurant. The MR scan ofthe brain reveals a 2 cm rounded cyst in the righttemporal lobe cortex and another 1.5 cm cyst in thesubarachnoid space over the left parietal lobe. Bothlesions are non-enhancing. A lumbar puncture yieldscolorless CSF under normal pressure. The protein andglucose are normal. There are 5 WBCs (4 monos, 1PMN). Which of the following conditions is most likely tobe present:A Metastatic adenocarcinomaB HIV encephalopathyC Left atrial mural thrombosisD CysticercosisE Hypertension

Page 257: usmle Board Review Step 1

257www.brain101.info

(D) CORRECT. This infection resultswhen man becomes the accidentalintermediate host for Taenia solium (porktapeworm). The larvae may lodge inmany organs but in the brain they areprimarily found in the subarachnoid spaceand the cortex.They become encystedand the cysts within the subarachnoidspace move around and can obstructCSF flow leading to life threateninghydrocephalus.

Page 258: usmle Board Review Step 1

258www.brain101.info

Several weeks after the appearance of anexpanding ring-like rash on her forearm, a 29-year-old female develops a stiff neck, left facialdroop, and chest pain. These problemsprobably developed as a consequence of:A The bite of a deer tickB Mercury poisoningC Taking a cephalosporinD Sexual intercourseE Systemic lupus erythematosus

Page 259: usmle Board Review Step 1

259www.brain101.info

(A) CORRECT. The findings suggesterythema migrans along with neurologicsequelae and pericarditis that result fromLyme disease.

Page 260: usmle Board Review Step 1

260www.brain101.info

A 61 year old male has had a chronic cough for manyyears as a result of smoking 2 packs of cigarettes perday. Recently, he has noted headaches. His physicianon neurologic exam can find no localizing signs. An MRIscan reveals a solitary 3.5 cm lesion that is located atthe grey-white junction in the posterior left frontal lobe.There is no ring enhancement. A stereotactic biopsy ofthis lesion is most likely to show:A An organizing abscessB Viral inclusionsC A plaque of demyelinationD Neuronal loss with gliosisE Metastatic carcinoma

Page 261: usmle Board Review Step 1

261www.brain101.info

(E) CORRECT. The location of the massat the grey-white junction is typical for ametastasis. Smoking increases the riskfor development of lung and renalcarcinomas that are the most commonsources for metastases to brain in males.

Page 262: usmle Board Review Step 1

262www.brain101.info

A 45 year old male complained of asevere headache. He was noted onphysical examination to havepapilledema on the right. He thenbecame obtunded and died. The grosspathologic finding seen here is most likelyto have been produced as aconsequence of:A Ocular melanomaB Berry aneurysm ruptureC Glioblastoma multiformeD Multiple sclerosisE Meningococcemia

Page 263: usmle Board Review Step 1

263www.brain101.info

C is CORRECT.A glioblastoma multiforme is a large, fast-growing neoplasm that can produceunilateral brain swelling and papilledema.

Page 264: usmle Board Review Step 1

264www.brain101.info

This lesion appeared on CT scan as a discreet mass beneath the dura andwas seen to compress the underlying cerebral hemisphere. The patient is a45 year old female who presented with headaches for the past month. Thebest diagnosis is:A MeningiomaB Astrocytoma, low gradeC EpendymomaD MetastasisE Tuberculoma

Page 265: usmle Board Review Step 1

265www.brain101.info

A is CORRECT.A meningioma is a benign neoplasm thatis slow-growing and can compress theunderlying brain without invasion. Theparasagittal region is a common location.

Page 266: usmle Board Review Step 1

266www.brain101.info

This 79 year old woman was driving her car when she had the sudden onset of a severeheadache. She pulled in to a service station and stopped the car. Then she slumped overthe wheel. She was taken to the emergency room, where she remained comatose and diedhours later. The most likely explanation for the gross appearance seen here is:A Glioblastoma multiformeB Thromboembolization with cerebral infarctionC Multiple sclerosisD Ruptured berry aneurysmE Huntington's disease

Page 267: usmle Board Review Step 1

267www.brain101.info

D is CORRECT.Rupture of a berry aneurysm involving thecircle of Willis and its branches is asudden event that produces hemorrhageinto the subarachnoid space at the baseof the brain.

Page 268: usmle Board Review Step 1

268www.brain101.info

A 60 year old male has the lesion shown here at autopsy. He had suddenly lost consciousness,and when he again became alert, he was unable to move his left arm or speak. Which of thefollowing underlying disease processes is he most likely to have to explain these findings:A Hypertension with chronic renal failureB Rheumatic heart disease with left atrial mural thrombosisC Chronic alcoholism with micronodular cirrhosisD Acquired immunodeficiency syndrome with low CD4 counE Papillary thyroid carcinoma with metastases to bone

Page 269: usmle Board Review Step 1

269www.brain101.info

B is CORRECT.Cerebral infarction can be the result ofthromboembolic disease. Manythromboemboli originate in the left side ofthe heart. Thromboembolic cerebralinfarctions can have hemorrhage, sincethe embolus may not completely occludethe artery.

Page 270: usmle Board Review Step 1

270www.brain101.info

For the past year, a 34 year old woman has been treated with highdose immunosuppressive therapy, including prednisone andcytoxan, for high grade lupus nephritis. She now presents withincreasing headache and decreased mentation for about a week. Acomplete blood count reveals that her WBC count is markedlydecreased. Based upon the microscopic appearance of the lesionseen here with Gomori methenamine silver stain (next slide), themost likely explanation for these findings is:A Progressive multifocal leukoencephalopathyB Multiple sclerosisC AspergillosisD Herpes simplex encephalitisE Cryptococcosis

Page 271: usmle Board Review Step 1

271www.brain101.info

Page 272: usmle Board Review Step 1

272www.brain101.info

C is CORRECT.The branching septate hyphae invadingthe vascular wall are typical for infectionwith Aspergillus.

Page 273: usmle Board Review Step 1

273www.brain101.info

A 75 year old female has had progressively worsening mental function, as indicated by her son whenhe brings his mother to see you. The son states that his mother is now frequently getting lost in theneighborhood. He relates that she cannot easily feed or dress herself. She often does not seem toknow who he his. After a year-long course in a nursing home, she dies from a respiratory infection. Thebrain at autopsy weighs 1025 gm and demonstrates frontal and parietal atrophy, with ex vacuoventricular dilation. The light microscopic appearance of the frontal cortex is seen here withBielschowsky silver stain. Which of the following statements best characterizes this process:A A bloodborne pathogen led to this illnessB An amyloid angiopathy can be presentC Inheritance of this disease occurs in an autosomal dominant fashionD A response to dopaminergic pharmacologic agents is often observedE Aluminum toxicity can explain these findings

Page 274: usmle Board Review Step 1

274www.brain101.info

B is CORRECT.Amyloid can be present within the senileplaques of Alzheimer's disease as well asin peripheral cerebral arteries. Sometimesthe amyloid angiopathy leads tointraparenchymal or subarachnoidhemorrhage.

Page 275: usmle Board Review Step 1

275www.brain101.info

Cerebral cortex that histologically shows numeroussenile plaques is most consistent with which of thefollowing histories:A A 48-year-old male with choreiform movementsB A 30-year-old female with sudden loss ofconsciousnessC A 44-year-old male with progressive muscularweaknessD A 40-year-old female with Down's syndromeE A 2-year-old boy with blindness and decreasedneurologic function

Page 276: usmle Board Review Step 1

276www.brain101.info

(D) CORRECT.The histologic finding is typical ofAlzheimer's disease. Persons with Down'ssyndrome who live to middle age oftendevelop Alzheimer's disease.

Page 277: usmle Board Review Step 1

277www.brain101.info

A 69-year-old male with a history of a remotemyocardial infarction is found at autopsy tohave a 4-cm diameter area of softening in theregion of the left middle cerebral arterydistribution. This is most consistent with:A VasculitisB Arterial embolizationC Venous thrombosisD HypertensionE Mycotic aneurysm

Page 278: usmle Board Review Step 1

278www.brain101.info

(B) CORRECT. The appearance of theinfarction in the major blood flowdistribution and the previous history ofheart disease suggests embolic disease.

Page 279: usmle Board Review Step 1

279www.brain101.info

A 50-year-old female has had right-sidedheadaches for 5 years, but recently noted mildweakness in her right hand. A CT scan shows awell circumscribed lateral mass compressingthe right hemisphere at the frontal-parietaljunction. This is probably a:A MedulloblastomaB Metastatic carcinomaC SchwannomaD Glioblastoma multiformeE Meningioma

Page 280: usmle Board Review Step 1

280www.brain101.info

(E) CORRECT. This is a slow-growingdural mass compressing the brain.

Page 281: usmle Board Review Step 1

281www.brain101.info

A 65-year-old male has been healthy all his lifeuntil a sudden seizure. Neurologic exam revealsno focal abnormalities. A CT scan reveals apoorly demarcated large mass with centralnecrosis in the right frontal lobe. The most likelydiagnosis is:A Glioblastoma multiformeB MedulloblastomaC Low grade astrocytomaD MeningiomaE Choroid plexus tumor

Page 282: usmle Board Review Step 1

282www.brain101.info

(A) CORRECT. High grade gliomas aremost likely to occur in adults and in thecerebral hemispheres. They are oftenlarge and infiltrative.

Page 283: usmle Board Review Step 1

283www.brain101.info

True statements regarding intracranial berryaneurysms include all of the following EXCEPT:A They are present at birth.B Subarachnoid hemorrhage could result fromrupture.C They can be associated with dominantpolycystic kidney disease.D Rupture is probably not associated withsystemic hypertension.E Intraparenchymal hemorrhage could resultfrom rupture.

Page 284: usmle Board Review Step 1

284www.brain101.info

(A) CORRECT. Although the arterialmedial weakness was present, theaneurysm itself developed over time.

Page 285: usmle Board Review Step 1

285www.brain101.info

A 52-year-old male with chronic respiratorydifficulty dies from bronchopneumonia. Atautopsy, the anterior spinal nerve roots areatrophic, and spinal cord anterior horns showneuronal loss with gliosis. These findings aremost consistent with:A Gullain-Barre syndromeB PoliomyelitisC Rabies infectionD BotulismE Amyotrophic lateral sclerosis

Page 286: usmle Board Review Step 1

286www.brain101.info

(B) CORRECT. Poliovirus infection leadsto destruction of anterior horn cells. Theanterior (motor) spinal nerve roots thenatrophy.

Page 287: usmle Board Review Step 1

287www.brain101.info

A 78-year-old male who suffers ischemicinjury with cerebral infarction most likelyhas which of the following histopathologicfindings:A Gangrenous necrosisB Liquefactive necrosisC Coagulative necrosisD Caseous necrosisE Fat necrosis

Page 288: usmle Board Review Step 1

288www.brain101.info

(B) CORRECT. The brain has a high lipidcontent and typically undergoesliquefaction with ischemic injury.

Page 289: usmle Board Review Step 1

289www.brain101.info

During a fight at the Beacon Club in Casper,Wyoming, a patron is knocked backwards off abarstool and lands on the back of his head.Which of the following pathologic findings ismost likely to be present as a consequence ofthis injury:A Occipital lobe contusionsB Subarachnoid hemorrhageC Inferior frontal lobe contusionsD Anterior pituitary necrosisE Skull fracture with epidural hematoma

Page 290: usmle Board Review Step 1

290www.brain101.info

(C) CORRECT. This is the location for acontracoup injury following a fallbackwards.

Page 291: usmle Board Review Step 1

291www.brain101.info

The presence of a neural tube defect issuggested most strongly by which of thefollowing findings:A Decreased maternal serum alpha-fetoproteinB MicrocephalyC HydranencephalyD Spina bifidaE Polyhydramnios

Page 292: usmle Board Review Step 1

292www.brain101.info

(D) CORRECT. This suggests a possibleneural tube defect.

Page 293: usmle Board Review Step 1

293www.brain101.info

Progressive spastic paraparesis, opticnerve atrophy, sensory ataxia, andmarked paresthesias of the legs are mostcharacteristic of a 44-year-old male with:A Chronic alcoholismB Down's syndromeC Pernicious anemiaD Lead poisoningE Diabetes mellitus, type I

Page 294: usmle Board Review Step 1

294www.brain101.info

(C) CORRECT. The subacute combineddegeneration of the spinal cord (posteriorand lateral white columns) from B12deficiency leads to these findings.

Page 295: usmle Board Review Step 1

295www.brain101.info

A 54-year-old female has a lumbar puncture performed.The cerebrospinal fluid (CSF) opening pressure is 220mm H2O, the CSF protein 60 mg/dl, and the CSFglucose 75 mg/dl (serum glucose 105 mg/dl). Biopsy ofa 3-cm right parietal mass reveals gliosis and fibrosiswith necrosis, neutrophils, and lymphocytes. Thesefindings suggest:A Glioblastoma multiformeB Herpes simplex type II encephalitisC Vascular malformationD Subacute infarctionE Cerebral abscess

Page 296: usmle Board Review Step 1

296www.brain101.info

(E) CORRECT. The necrosis with fibrosisis typical for an abscess with anorganizing wall. Neovascularizationaround the organization leads to edema.

Page 297: usmle Board Review Step 1

297www.brain101.info

A 50-year-old Vietnam veteran had a history ofparesthesias, difficulty moving one or more extremities,loss of sensation, and ataxia over 22 years. Theseproblems would come and go, but he eventuallydeveloped paraplegia and incontinence. Which of thefollowing findings best explains his history:A Parasaggittal meningiomaB Scattered plaques of demyelinationC Shrapnel in spinal cordD Wernicke's diseaseE Progressive multifocal leuko- encephalopathy

Page 298: usmle Board Review Step 1

298www.brain101.info

(B) CORRECT. The history is mostconsistent with multiple sclerosis.

Page 299: usmle Board Review Step 1

299www.brain101.info

Which of the following statements concerningan epidural hematoma is most appropriate:A It is accompanied by a skull fracture.B The bleeding is of venous origin.C Onset of symptoms is delayed followingvascular rupture.D A fluctuating level of consciousness isevidenced by the patient.E It can occur as a result of rupture of a mycoticaneuysm.

Page 300: usmle Board Review Step 1

300www.brain101.info

(A) CORRECT. An epidural hematoma isalmost always preceded by a skullfracture that results in a tear of the middlemeningeal artery.

Page 301: usmle Board Review Step 1

301www.brain101.info

Following a mild upper respiratory flu-like illness, a 47year old male develops a rapidly ascending paralysis. Aweek later he is hospitalized and requires intubationwith mechanical ventilation. Lumbar puncture yieldsclear CSF under normal pressure with a slightlyelevated protein, but no red blood cells and only 3mononuclear cells. He gradually improves over the nextcouple of weeks. He most likely has:A Multiple sclerosisB Amyotrophic lateral sclerosisC Huntington's diseaseD Guillain-Barré syndromeE Werdnig-Hoffman disease

Page 302: usmle Board Review Step 1

302www.brain101.info

(D) CORRECT. About 3/4 of patientshave an antecedent viral infection. Thisdisease is thought to be the result of animmunologic response..

Page 303: usmle Board Review Step 1

303www.brain101.info

Loss of all somatic sensation on the rightside of the tongue is likely with damageto:A. the right hypoglossal nerveB. the left hypoglossal nerveC. the right facial nerveD. the right trigeminal nerve

Page 304: usmle Board Review Step 1

304www.brain101.info

D. the right trigeminal nerve

Page 305: usmle Board Review Step 1

305www.brain101.info

Loss of sensation on the little finger (yourpinkie) is likely to result from damage toprimary afferent fibers that enter thespinal cord at:A. L1B.C8C.C1D.T2

Page 306: usmle Board Review Step 1

306www.brain101.info

B. C8

Page 307: usmle Board Review Step 1

307www.brain101.info

Axons in the medial lemniscus cross themidline in:A. the ventral white commissureB. the pyramidal decussationC. the decussation of the dorsal columnsD. the internal arcuate fibers

Page 308: usmle Board Review Step 1

308www.brain101.info

D. the internal arcuate fibers

Page 309: usmle Board Review Step 1

309www.brain101.info

Fine touch and proprioception informationtravels through the ____ of the spinalcord.A. anterior funiculusB. lateral funiculusC. posterior funiculus

Page 310: usmle Board Review Step 1

310www.brain101.info

C. posterior funiculus

Page 311: usmle Board Review Step 1

311www.brain101.info

Fasciculus Gracilis contains axons thatmainly synapse in:A. the ipsilateral nucleus gracilisB. the contralateral ventral posteriorlateral nucleus of the thalamusC. the contralateral nucleus gracilisD. the contralateral dorsal horn

Page 312: usmle Board Review Step 1

312www.brain101.info

A. the ipsilateral nucleus gracilis

Page 313: usmle Board Review Step 1

313www.brain101.info

Axons which synapse in nucleus cuneatishave their cell bodies in:A. cervical or thoracic dorsal hornB. lumbar or sacral dorsal hornC. cervical or thoracic dorsal root gangliaD. lumbar or sacral dorsal root ganglia

Page 314: usmle Board Review Step 1

314www.brain101.info

C. cervical or thoracic dorsal root ganglia

Page 315: usmle Board Review Step 1

315www.brain101.info

Damage to the posterior funiculus at T10 on theleft side is likely to produceA. inability to detect a vibrating stimulus when itis placed on the right kneeB. loss of pain and temperature below thelesion on the contralateral sideC. inability to detect the stroke of an artist’sbrush on the bottom of the left footD. loss of the Babinski reflex below the lesionon the ipsilateral side

Page 316: usmle Board Review Step 1

316www.brain101.info

C. inability to detect the stroke of anartist’s brush on the bottom of the left foot

Page 317: usmle Board Review Step 1

317www.brain101.info

The medial lemniscus carries fine touchand proprioception information as far asA. the medullaB. the ponsC. the thalamusD. the somatosensory cortex

Page 318: usmle Board Review Step 1

318www.brain101.info

C. the thalamus

Page 319: usmle Board Review Step 1

319www.brain101.info

Damage to the most ventral region of themedial lemniscus in the middle medulla shouldresult in:A. loss of conscious proprioception in thecontralateral ankle jointB. loss of fine touch in the ipsilateral footC. loss of fine touch in the contralateral handD. loss of proprioception in the ipsilateral hipjoint

Page 320: usmle Board Review Step 1

320www.brain101.info

A. loss of conscious proprioception in thecontralateral ankle joint

Page 321: usmle Board Review Step 1

321www.brain101.info

In the sensory homunculus of thepostcentral gyrus, which area of the bodyis represented most laterally?A. the feetB. the noseC. the tongueD. the forehead

Page 322: usmle Board Review Step 1

322www.brain101.info

C. the tongue

Page 323: usmle Board Review Step 1

323www.brain101.info

When there is damage to the spinal cordthere is often an ipsilateral loss of painand temperature:A. at and below the level of the lesionB. below the level of the lesionC. above the level of the lesionD. at the level of the lesion

Page 324: usmle Board Review Step 1

324www.brain101.info

D. at the level of the lesion

Page 325: usmle Board Review Step 1

325www.brain101.info

The main neurotransmitter that isreleased both peripherally and centrallyby primary pain afferent fibers is:A. gamma amino butyric acidB. glutamateC. acetylcholineD. Substance P

Page 326: usmle Board Review Step 1

326www.brain101.info

D. Substance P

Page 327: usmle Board Review Step 1

327www.brain101.info

The reason the ALF is not the same asthe spinothalamic tract is:A. it starts in the spinal cord, but doesn’tterminate in the thalamusB. it is a fasiculus not a tractC. it is an Alien not a tractD. it contains the spinothalamic tract butalso several other tracts

Page 328: usmle Board Review Step 1

328www.brain101.info

D. it contains the spinothalamic tract butalso several other tracts

Page 329: usmle Board Review Step 1

329www.brain101.info

What type of fibers travel in thedorsolateral fasciculus (Lissaeur’s Tract)?A. primary pain afferentsB. primary touch afferentsC. secondary pain afferentsD. primary pain and touch afferents

Page 330: usmle Board Review Step 1

330www.brain101.info

A. primary pain afferents

Page 331: usmle Board Review Step 1

331www.brain101.info

The cell bodies of the axons in Lissaeur’sTract (the dorsolateral fasciculus) arelocated in:A. the dorsal root gangliaB. the dorsal hornC. the ventral hornD. the Gasserian ganglia

Page 332: usmle Board Review Step 1

332www.brain101.info

A. the dorsal root ganglia

Page 333: usmle Board Review Step 1

333www.brain101.info

Pain information that reaches the postcentralgyrus is carried by axons that originate inneurons located in:A. nucleus gracilis or cuneatisB. ventral posterior lateral nucleus of thethalamusC. dorsal horn of the spinal cordD. medial and intralaminar nuclei of thethalamus

Page 334: usmle Board Review Step 1

334www.brain101.info

B. ventral posterior lateral nucleus of thethalamus

Page 335: usmle Board Review Step 1

335www.brain101.info

In syringomyelia, the central canal of the spinalcord enlarges dramatically, especially atcervical levels. A likely outcome is:A. difficulty distinguishing coins in the patient’spocketB. ignoring a cut or burn of the hand, leading toinfectionC. loss of pain perception bilaterally at all levelsof the spinal cordD. paralysis of the upper and lower limbs with adecrease in tone.

Page 336: usmle Board Review Step 1

336www.brain101.info

B. ignoring a cut or burn of the hand,leading to infection

Page 337: usmle Board Review Step 1

337www.brain101.info

Patients with blockage of the middle cerebralartery often retain pain sensation because:A. the somatosensory cortex receives it’s bloodsupply from the anterior cerebral arteryB. blockage of the MCA may not damage thethalamusC. the somatosensory cortex does not processpain informationD. blockage of the MCA does not damage thecingulate cortex

Page 338: usmle Board Review Step 1

338www.brain101.info

D. blockage of the MCA does not damagethe cingulate cortex

Page 339: usmle Board Review Step 1

339www.brain101.info

What muscle is most responsible forabducting the eye?A. superior rectusB. superior obliqueC. inferior obliqueD. lateral rectus

Page 340: usmle Board Review Step 1

340www.brain101.info

D. lateral rectus

Page 341: usmle Board Review Step 1

341www.brain101.info

The spinal accessory nerve exits the skullthrough:A. the foramen magnumB. the internal auditory meatusC. foramen spinosumD. the jugular foramen

Page 342: usmle Board Review Step 1

342www.brain101.info

D. the jugular foramen

Page 343: usmle Board Review Step 1

343www.brain101.info

If a patient cannot look to the left with hisleft eye, then there may well be damageto:A. the abducens nerveB. the trochlear nerveC. the oculomotor nerve

Page 344: usmle Board Review Step 1

344www.brain101.info

A. the abducens nerve

Page 345: usmle Board Review Step 1

345www.brain101.info

The facial nerve closes the eye lid whilethe ___ nerve opens it.A. 5thB. 7thC. 4thD. 3rd

Page 346: usmle Board Review Step 1

346www.brain101.info

D. 3rd

Page 347: usmle Board Review Step 1

347www.brain101.info

The hypoglossal nerve exits thebrainstem at:A. the preolivary sulcusB. the postolivary sulcusC. the pontomedullary junctionD. the obex

Page 348: usmle Board Review Step 1

348www.brain101.info

A. the preolivary sulcus

Page 349: usmle Board Review Step 1

349www.brain101.info

When the eye is adducted, which muscleelevates the eye?A. superior rectusB. superior obliqueC. inferior obliqueD. inferior rectus

Page 350: usmle Board Review Step 1

350www.brain101.info

C. inferior oblique

Page 351: usmle Board Review Step 1

351www.brain101.info

Damage to the ventral midbrain is likely toproduce all of the following symptoms EXCEPT:A. difficulty in depressing the ipsilateral eyewhen it is aDductedB. a drooping eyelidC. a dilated pupilD. difficulty in depressing the ipsilateral eyewhen it is aBducted

Page 352: usmle Board Review Step 1

352www.brain101.info

A. difficulty in depressing the ipsilateraleye when it is aDducted

Page 353: usmle Board Review Step 1

353www.brain101.info

Visceral sensory fibers in the vagus nerveterminate mainly in:A. nucleus cuneatisB. nucleus ambiguousC. the nodose gangliaD. the solitary nucleus

Page 354: usmle Board Review Step 1

354www.brain101.info

D. the solitary nucleus

Page 355: usmle Board Review Step 1

355www.brain101.info

If your gag reflex was absent AND whenyour uvula was stimulated you had noconscious sensation of gagging, then it islikely that there is damage to at least:A. the vagus nerveB. the glossopharyngeal nerveC. the facial nerveD. the hypoglossal nerve

Page 356: usmle Board Review Step 1

356www.brain101.info

B. the glossopharyngeal nerve

Page 357: usmle Board Review Step 1

www.brain101.info

Good Luck!